Difference between revisions of "1964 AHSME Problems"

(Created page with "== Problem 1== What is the value of <math>[\log_{10}(5\log_{10}100)]^2</math>? <math>\textbf{(A)}\ \log_{10}50 \qquad \textbf{(B)}\ 25\qquad \textbf{(C)}\ 10 \qquad \textbf{(D)...")
 
(See also)
 
(8 intermediate revisions by 4 users not shown)
Line 1: Line 1:
 +
{{AHSC 40 Problems
 +
|year = 1964
 +
}}
 
== Problem 1==
 
== Problem 1==
  
Line 6: Line 9:
 
\textbf{(B)}\ 25\qquad
 
\textbf{(B)}\ 25\qquad
 
\textbf{(C)}\ 10 \qquad
 
\textbf{(C)}\ 10 \qquad
\textbf{(D)}\ 2}\qquad
+
\textbf{(D)}\ 2\qquad
\textbf{(E)}\ 1 </math>   
+
\textbf{(E)}\ 1 </math>   
  
 
[[1964 AHSME Problems/Problem 1|Solution]]
 
[[1964 AHSME Problems/Problem 1|Solution]]
Line 17: Line 20:
 
<math>\textbf{(A)}\ \text{a parabola} \qquad
 
<math>\textbf{(A)}\ \text{a parabola} \qquad
 
\textbf{(B)}\ \text{an ellipse} \qquad
 
\textbf{(B)}\ \text{an ellipse} \qquad
\textbf{(C)}\ \text{a pair of straight lines}\qquad
+
\textbf{(C)}\ \text{a pair of straight lines}\qquad \\
\textbf{(D)}\ \text{a point}}\qquad
+
\textbf{(D)}\ \text{a point}\qquad
\textbf{(E)}\ \text{None of these}} </math>     
+
\textbf{(E)}\ \text{None of these}</math>     
  
 
[[1964 AHSME Problems/Problem 2|Solution]]
 
[[1964 AHSME Problems/Problem 2|Solution]]
Line 31: Line 34:
 
\textbf{(B)}\ 2u \qquad
 
\textbf{(B)}\ 2u \qquad
 
\textbf{(C)}\ 3u \qquad
 
\textbf{(C)}\ 3u \qquad
\textbf{(D)}\ v }\qquad
+
\textbf{(D)}\ v \qquad
\textbf{(E)}\ 2v }</math>     
+
\textbf{(E)}\ 2v </math>     
  
 
[[1964 AHSME Problems/Problem 3|Solution]]
 
[[1964 AHSME Problems/Problem 3|Solution]]
Line 48: Line 51:
 
\textbf{(C)}\ 1 \qquad
 
\textbf{(C)}\ 1 \qquad
 
\textbf{(D)}\ \frac{x^2+y^2}{xy}\qquad
 
\textbf{(D)}\ \frac{x^2+y^2}{xy}\qquad
\textbf{(E)}\ \frac{x^2+y^2}{2xy}}  </math>  
+
\textbf{(E)}\ \frac{x^2+y^2}{2xy}  </math>  
  
 
[[1964 AHSME Problems/Problem 4|Solution]]
 
[[1964 AHSME Problems/Problem 4|Solution]]
Line 56: Line 59:
 
If <math>y</math> varies directly as <math>x</math>, and if <math>y=8</math> when <math>x=4</math>, the value of <math>y</math> when <math>x=-8</math> is:
 
If <math>y</math> varies directly as <math>x</math>, and if <math>y=8</math> when <math>x=4</math>, the value of <math>y</math> when <math>x=-8</math> is:
  
<math>\textbf{(A)}\ -16} \qquad
+
<math>\textbf{(A)}\ -16 \qquad
 
\textbf{(B)}\ -4 \qquad
 
\textbf{(B)}\ -4 \qquad
 
\textbf{(C)}\ -2 \qquad
 
\textbf{(C)}\ -2 \qquad
\textbf{(D)}\ 4k, k= \pm1, \pm2, \dots}
+
\textbf{(D)}\ 4k, k= \pm1, \pm2, \dots \qquad \\
\qquad
+
\textbf{(E)}\ 16k, k=\pm1,\pm2,\dots     </math>
\textbf{(E)}\ 16k, k=\pm1,\pm2,\dots }    </math>
 
  
 
[[1964 AHSME Problems/Problem 5|Solution]]
 
[[1964 AHSME Problems/Problem 5|Solution]]
Line 72: Line 74:
 
\textbf{(B)}\ -13\frac{1}{2} \qquad
 
\textbf{(B)}\ -13\frac{1}{2} \qquad
 
\textbf{(C)}\ 12\qquad
 
\textbf{(C)}\ 12\qquad
\textbf{(D)}\ 13\frac{1}{2}}\qquad
+
\textbf{(D)}\ 13\frac{1}{2}\qquad
\textbf{(E)}\ 27 } </math>   
+
\textbf{(E)}\ 27 </math>   
  
 
[[1964 AHSME Problems/Problem 6|Solution]]
 
[[1964 AHSME Problems/Problem 6|Solution]]
Line 93: Line 95:
 
== Problem 8==
 
== Problem 8==
  
The smaller root of the equation <math>\left(x-\frac{3}{4}\right)\left(x-\frac{3}{4}\right)+\left(x-\frac{3}{4}\right)\left(x-\frac{1}{2}\right) =0</math> is:
+
The smaller root of the equation  
 +
<math>\left(x-\frac{3}{4}\right)\left(x-\frac{3}{4}\right)+\left(x-\frac{3}{4}\right)\left(x-\frac{1}{2}\right) =0</math> is:
  
 
<math>\textbf{(A)}\ -\frac{3}{4}\qquad
 
<math>\textbf{(A)}\ -\frac{3}{4}\qquad
 
\textbf{(B)}\ \frac{1}{2}\qquad
 
\textbf{(B)}\ \frac{1}{2}\qquad
 
\textbf{(C)}\ \frac{5}{8}\qquad
 
\textbf{(C)}\ \frac{5}{8}\qquad
\textbf{(D)}\ \frac{3}{4}}\qquad
+
\textbf{(D)}\ \frac{3}{4}\qquad
\textbf{(E)}\ 1 }
+
\textbf{(E)}\ 1 </math>
  
 
[[1964 AHSME Problems/Problem 8|Solution]]
 
[[1964 AHSME Problems/Problem 8|Solution]]
Line 105: Line 108:
 
== Problem 9==
 
== Problem 9==
  
A jobber buys an article at &#036;24 less </math>12\frac{1}{2}<math> %. He then wishes to sell the article at a gain of </math>33\frac{1}{3}<math> % of his cost  
+
A jobber buys an article at <math>\$24</math> less <math>12\frac{1}{2}\%</math>. He then wishes to sell the article at a gain of <math>33\frac{1}{3}\%</math> of his cost  
after allowing a 20% discount on his marked price. At what price, in dollars, should the article be marked?
+
after allowing a <math>20\%</math> discount on his marked price. At what price, in dollars, should the article be marked?
  
</math>\textbf{(A)}\ 25.20 \qquad
+
<math>\textbf{(A)}\ 25.20 \qquad
 
\textbf{(B)}\ 30.00 \qquad
 
\textbf{(B)}\ 30.00 \qquad
 
\textbf{(C)}\ 33.60 \qquad
 
\textbf{(C)}\ 33.60 \qquad
\textbf{(D)}\ 40.00 }\qquad
+
\textbf{(D)}\ 40.00 \qquad
\textbf{(E)}\ \text{none of these}} <math>
+
\textbf{(E)}\ \text{none of these} </math>
  
 
[[1964 AHSME Problems/Problem 9|Solution]]
 
[[1964 AHSME Problems/Problem 9|Solution]]
Line 118: Line 121:
 
== Problem 10==
 
== Problem 10==
  
Given a square side of length </math>s<math>. On a diagonal as base a triangle with three unequal sides is constructed so that its area
+
Given a square side of length <math>s</math>. On a diagonal as base a triangle with three unequal sides is constructed so that its area
 
equals that of the square. The length of the altitude drawn to the base is:
 
equals that of the square. The length of the altitude drawn to the base is:
  
</math>\textbf{(A)}\ s\sqrt{2} \qquad
+
<math>\textbf{(A)}\ s\sqrt{2} \qquad
 
\textbf{(B)}\ s/\sqrt{2} \qquad
 
\textbf{(B)}\ s/\sqrt{2} \qquad
 
\textbf{(C)}\ 2s \qquad
 
\textbf{(C)}\ 2s \qquad
\textbf{(D)}\ 2\sqrt{s} }\qquad
+
\textbf{(D)}\ 2\sqrt{s} \qquad
\textbf{(E)}\ 2/\sqrt{s}} <math>
+
\textbf{(E)}\ 2/\sqrt{s}</math>
  
 
[[1964 AHSME Problems/Problem 10|Solution]]
 
[[1964 AHSME Problems/Problem 10|Solution]]
Line 131: Line 134:
 
== Problem 11==
 
== Problem 11==
  
Given </math>2^x=8^{y+1}<math> and </math>9^y=3^{x-9}<math>, find the value of </math>x+y<math>
+
Given <math>2^x=8^{y+1}</math> and <math>9^y=3^{x-9}</math>, find the value of <math>x+y</math>
  
</math>\textbf{(A)}\ 18 \qquad
+
<math>\textbf{(A)}\ 18 \qquad
 
\textbf{(B)}\ 21 \qquad
 
\textbf{(B)}\ 21 \qquad
 
\textbf{(C)}\ 24 \qquad
 
\textbf{(C)}\ 24 \qquad
\textbf{(D)}\ 27 }\qquad
+
\textbf{(D)}\ 27 \qquad
\textbf{(E)}\ 30 } <math>     
+
\textbf{(E)}\ 30 </math>     
  
 
[[1964 AHSME Problems/Problem 11|Solution]]
 
[[1964 AHSME Problems/Problem 11|Solution]]
Line 143: Line 146:
 
== Problem 12==
 
== Problem 12==
  
Which of the following is the negation of the statement: For all </math>x<math> of a certain set, </math>x^2>0<math>?
+
Which of the following is the negation of the statement: For all <math>x</math> of a certain set, <math>x^2>0</math>?
  
</math>\textbf{(A)}\ \text{For all x}, x^2 < 0\qquad
+
<math>\textbf{(A)}\ \text{For all x}, x^2 < 0\qquad
 
\textbf{(B)}\ \text{For all x}, x^2 \le 0\qquad
 
\textbf{(B)}\ \text{For all x}, x^2 \le 0\qquad
\textbf{(C)}\ \text{For no x}, x^2>0\qquad
+
\textbf{(C)}\ \text{For no x}, x^2>0\qquad \\
\textbf{(D)}\ \text{For some x}, x^2>0 }\qquad
+
\textbf{(D)}\ \text{For some x}, x^2>0\qquad
\textbf{(E)}\ \text{For some x}, x^2 \le 0}}   <math>  
+
\textbf{(E)}\ \text{For some x}, x^2 \le 0    </math>  
  
 
[[1964 AHSME Problems/Problem 12|Solution]]
 
[[1964 AHSME Problems/Problem 12|Solution]]
Line 155: Line 158:
 
== Problem 13==
 
== Problem 13==
  
A circle is inscribed in a triangle with side lengths </math>8, 13<math>, and </math>17<math>. Let the segments of the side of length </math>8<math>,  
+
A circle is inscribed in a triangle with side lengths <math>8, 13</math>, and <math>17</math>. Let the segments of the side of length <math>8</math>,  
made by a point of tangency, be </math>r<math> and </math>s<math>, with </math>r<s<math>. What is the ratio </math>r:s<math>?  
+
made by a point of tangency, be <math>r</math> and <math>s</math>, with <math>r<s</math>. What is the ratio <math>r:s</math>?  
  
</math>\textbf{(A)}\ 1:3 \qquad
+
<math>\textbf{(A)}\ 1:3 \qquad
 
\textbf{(B)}\ 2:5 \qquad
 
\textbf{(B)}\ 2:5 \qquad
 
\textbf{(C)}\ 1:2 \qquad
 
\textbf{(C)}\ 1:2 \qquad
\textbf{(D)}\ 2:3 }\qquad
+
\textbf{(D)}\ 2:3 \qquad
\textbf{(E)}\ 3:4 <math>   
+
\textbf{(E)}\ 3:4   </math>   
  
 
[[1964 AHSME Problems/Problem 13|Solution]]
 
[[1964 AHSME Problems/Problem 13|Solution]]
Line 168: Line 171:
 
== Problem 14==
 
== Problem 14==
  
A farmer bought </math>749<math> sheep. He sold </math>700<math> of them for the price paid for the </math>749<math> sheep.  
+
A farmer bought <math>749</math> sheep. He sold <math>700</math> of them for the price paid for the <math>749</math> sheep.  
The remaining </math>49<math> sheep were sold at the same price per head as the other </math>700<math>.  
+
The remaining <math>49</math> sheep were sold at the same price per head as the other <math>700</math>.  
 
Based on the cost, the percent gain on the entire transaction is:
 
Based on the cost, the percent gain on the entire transaction is:
  
</math>\textbf{(A)}\ 6.5 \qquad
+
<math>\textbf{(A)}\ 6.5 \qquad
 
\textbf{(B)}\ 6.75 \qquad
 
\textbf{(B)}\ 6.75 \qquad
 
\textbf{(C)}\ 7 \qquad
 
\textbf{(C)}\ 7 \qquad
\textbf{(D)}\ 7.5 }\qquad
+
\textbf{(D)}\ 7.5 \qquad
\textbf{(E)}\ 8 } <math>   
+
\textbf{(E)}\ 8  </math>   
  
 
[[1964 AHSME Problems/Problem 14|Solution]]
 
[[1964 AHSME Problems/Problem 14|Solution]]
Line 182: Line 185:
 
== Problem 15==
 
== Problem 15==
  
A line through the point </math>(-a,0)<math> cuts from the second quadrant a triangular region with area </math>T<math>. The equation of the line is:
+
A line through the point <math>(-a,0)</math> cuts from the second quadrant a triangular region with area <math>T</math>. The equation of the line is:
  
</math>\textbf{(A)}\ 2Tx+a^2y+2aT=0 \qquad
+
<math>\textbf{(A)}\ 2Tx+a^2y+2aT=0 \qquad
 
\textbf{(B)}\ 2Tx-a^2y+2aT=0 \qquad
 
\textbf{(B)}\ 2Tx-a^2y+2aT=0 \qquad
\textbf{(C)}\ 2Tx+a^2y-2aT=0 \qquad
+
\textbf{(C)}\ 2Tx+a^2y-2aT=0 \qquad \\
\textbf{(D)}\ 2Tx-a^2y-2aT=0 }\qquad
+
\textbf{(D)}\ 2Tx-a^2y-2aT=0 \qquad
\textbf{(E)}\ \text{none of these} }  <math>   
+
\textbf{(E)}\ \text{none of these}  </math>   
  
 
[[1964 AHSME Problems/Problem 15|Solution]]
 
[[1964 AHSME Problems/Problem 15|Solution]]
Line 194: Line 197:
 
== Problem 16==
 
== Problem 16==
  
Let </math>f(x)=x^2+3x+2<math> and let </math>S<math> be the set of integers </math>\{0, 1, 2, \dots , 25 \}<math>.  
+
Let <math>f(x)=x^2+3x+2</math> and let <math>S</math> be the set of integers <math>\{0, 1, 2, \dots , 25 \}</math>.  
The number of members </math>s<math> of </math>S<math> such that </math>f(s)<math> has remainder zero when divided by </math>6<math> is:
+
The number of members <math>s</math> of <math>S</math> such that <math>f(s)</math> has remainder zero when divided by <math>6</math> is:
  
</math>\textbf{(A)}\ 25\qquad
+
<math>\textbf{(A)}\ 25\qquad
 
\textbf{(B)}\ 22\qquad
 
\textbf{(B)}\ 22\qquad
 
\textbf{(C)}\ 21\qquad
 
\textbf{(C)}\ 21\qquad
\textbf{(D)}\ 18 }\qquad
+
\textbf{(D)}\ 18 \qquad
\textbf{(E)}\ 17 }    <math>
+
\textbf{(E)}\ 17   </math>
  
 
[[1964 AHSME Problems/Problem 16|Solution]]
 
[[1964 AHSME Problems/Problem 16|Solution]]
Line 207: Line 210:
 
== Problem 17==
 
== Problem 17==
  
Given the distinct points </math>P(x_1, y_1), Q(x_2, y_2)<math> and </math>R(x_1+x_2, y_1+y_2)<math>.  
+
Given the distinct points <math>P(x_1, y_1), Q(x_2, y_2)</math> and <math>R(x_1+x_2, y_1+y_2)</math>.  
Line segments are drawn connecting these points to each other and to the origin </math>0<math>.  
+
Line segments are drawn connecting these points to each other and to the origin <math>O</math>.  
Of the three possibilities: (1) parallelogram (2) straight line (3) trapezoid, figure </math>OPRQ<math>,  
+
Of the three possibilities: (1) parallelogram (2) straight line (3) trapezoid, figure <math>OPRQ</math>,  
depending upon the location of the points </math>P, Q<math>, and </math>R<math>, can be:
+
depending upon the location of the points <math>P, Q</math>, and <math>R</math>, can be:
  
</math>\textbf{(A)}\ \text{(1) only}\qquad
+
<math>\textbf{(A)}\ \text{(1) only}\qquad
 
\textbf{(B)}\ \text{(2) only}\qquad
 
\textbf{(B)}\ \text{(2) only}\qquad
 
\textbf{(C)}\ \text{(3) only}\qquad
 
\textbf{(C)}\ \text{(3) only}\qquad
 
\textbf{(D)}\ \text{(1) or (2) only}\qquad
 
\textbf{(D)}\ \text{(1) or (2) only}\qquad
\textbf{(E)}\ \text{all three} <math>   
+
\textbf{(E)}\ \text{all three} </math>   
  
 
[[1964 AHSME Problems/Problem 17|Solution]]
 
[[1964 AHSME Problems/Problem 17|Solution]]
Line 222: Line 225:
 
== Problem 18==
 
== Problem 18==
  
Let </math>n<math> be the number of pairs of values of </math>b<math> and </math>c<math> such that </math>3x+by+c=0<math> and </math>cx-2y+12=0<math> have the same graph. Then </math>n<math> is:
+
Let <math>n</math> be the number of pairs of values of <math>b</math> and <math>c</math> such that <math>3x+by+c=0</math> and <math>cx-2y+12=0</math> have the same graph. Then <math>n</math> is:
  
</math>\textbf{(A)}\ 0\qquad
+
<math>\textbf{(A)}\ 0\qquad
 
\textbf{(B)}\ 1\qquad
 
\textbf{(B)}\ 1\qquad
 
\textbf{(C)}\ 2\qquad
 
\textbf{(C)}\ 2\qquad
 
\textbf{(D)}\ \text{finite but more than 2}\qquad
 
\textbf{(D)}\ \text{finite but more than 2}\qquad
\textbf{(E)}\ \infty <math>   
+
\textbf{(E)}\ \infty </math>   
  
 
[[1964 AHSME Problems/Problem 18|Solution]]
 
[[1964 AHSME Problems/Problem 18|Solution]]
Line 234: Line 237:
 
== Problem 19==
 
== Problem 19==
  
If </math>2x-3y-z=0<math> and </math>x+3y-14z=0, z \neq 0<math>, the numerical value of </math>\frac{x^2+3xy}{y^2+z^2}<math> is:
+
If <math>2x-3y-z=0</math> and <math>x+3y-14z=0, z \neq 0</math>, the numerical value of <math>\frac{x^2+3xy}{y^2+z^2}</math> is:
  
</math>\textbf{(A)}\ 7\qquad
+
<math>\textbf{(A)}\ 7\qquad
 
\textbf{(B)}\ 2\qquad
 
\textbf{(B)}\ 2\qquad
 
\textbf{(C)}\ 0\qquad
 
\textbf{(C)}\ 0\qquad
 
\textbf{(D)}\ -20/17\qquad
 
\textbf{(D)}\ -20/17\qquad
\textbf{(E)}\ -2  <math>   
+
\textbf{(E)}\ -2  </math>   
  
 
[[1964 AHSME Problems/Problem 19|Solution]]
 
[[1964 AHSME Problems/Problem 19|Solution]]
Line 246: Line 249:
 
== Problem 20==
 
== Problem 20==
  
The sum of the numerical coefficients of all the terms in the expansion of </math>(x-2y)^{18}<math> is:
+
The sum of the numerical coefficients of all the terms in the expansion of <math>(x-2y)^{18}</math> is:
  
</math>\textbf{(A)}\ 0\qquad
+
<math>\textbf{(A)}\ 0\qquad
 
\textbf{(B)}\ 1\qquad
 
\textbf{(B)}\ 1\qquad
 
\textbf{(C)}\ 19\qquad
 
\textbf{(C)}\ 19\qquad
 
\textbf{(D)}\ -1\qquad
 
\textbf{(D)}\ -1\qquad
\textbf{(E)}\ -19  <math>   
+
\textbf{(E)}\ -19  </math>   
  
 
[[1964 AHSME Problems/Problem 20|Solution]]
 
[[1964 AHSME Problems/Problem 20|Solution]]
Line 258: Line 261:
 
== Problem 21==
 
== Problem 21==
  
If </math>\log_{b^2}x+\log_{x^2}b=1, b>0, b \neq 1, x \neq 1<math>, then </math>x<math> equals:
+
If <math>\log_{b^2}x+\log_{x^2}b=1, b>0, b \neq 1, x \neq 1</math>, then <math>x</math> equals:
  
</math>\textbf{(A)}\ 1/b^2 \qquad
+
<math>\textbf{(A)}\ 1/b^2 \qquad
 
\textbf{(B)}\ 1/b \qquad
 
\textbf{(B)}\ 1/b \qquad
 
\textbf{(C)}\ b^2 \qquad
 
\textbf{(C)}\ b^2 \qquad
 
\textbf{(D)}\ b \qquad
 
\textbf{(D)}\ b \qquad
\textbf{(E)}\ \sqrt{b} <math>     
+
\textbf{(E)}\ \sqrt{b} </math>     
  
 
[[1964 AHSME Problems/Problem 21|Solution]]
 
[[1964 AHSME Problems/Problem 21|Solution]]
Line 270: Line 273:
 
== Problem 22==
 
== Problem 22==
  
Given parallelogram </math>ABCD<math> with </math>E<math> the midpoint of diagonal </math>BD<math>. Point </math>E<math> is connected to a point </math>F<math> in </math>DA<math> so that  
+
Given parallelogram <math>ABCD</math> with <math>E</math> the midpoint of diagonal <math>BD</math>. Point <math>E</math> is connected to a point <math>F</math> in <math>DA</math> so that  
</math>DF=\frac{1}{3}DA<math>. What is the ratio of the area of </math>\triangle DFE<math> to the area of quadrilateral </math>ABEF<math>?
+
<math>DF=\frac{1}{3}DA</math>. What is the ratio of the area of <math>\triangle DFE</math> to the area of quadrilateral <math>ABEF</math>?
  
</math>\textbf{(A)}\ 1:2 \qquad
+
<math>\textbf{(A)}\ 1:2 \qquad
 
\textbf{(B)}\ 1:3 \qquad
 
\textbf{(B)}\ 1:3 \qquad
 
\textbf{(C)}\ 1:5 \qquad
 
\textbf{(C)}\ 1:5 \qquad
 
\textbf{(D)}\ 1:6 \qquad
 
\textbf{(D)}\ 1:6 \qquad
\textbf{(E)}\ 1:7  <math>   
+
\textbf{(E)}\ 1:7  </math>   
  
 
[[1964 AHSME Problems/Problem 22|Solution]]
 
[[1964 AHSME Problems/Problem 22|Solution]]
Line 283: Line 286:
 
== Problem 23==
 
== Problem 23==
  
Two numbers are such that their difference, their sum, and their product are to one another as </math>1:7:24<math>. The product of the two numbers is:
+
Two numbers are such that their difference, their sum, and their product are to one another as <math>1:7:24</math>. The product of the two numbers is:
  
</math>\textbf{(A)}\ 6\qquad
+
<math>\textbf{(A)}\ 6\qquad
 
\textbf{(B)}\ 12\qquad
 
\textbf{(B)}\ 12\qquad
 
\textbf{(C)}\ 24\qquad
 
\textbf{(C)}\ 24\qquad
 
\textbf{(D)}\ 48\qquad
 
\textbf{(D)}\ 48\qquad
\textbf{(E)}\ 96    <math>  
+
\textbf{(E)}\ 96    </math>  
  
 
[[1964 AHSME Problems/Problem 23|Solution]]
 
[[1964 AHSME Problems/Problem 23|Solution]]
Line 295: Line 298:
 
== Problem 24==
 
== Problem 24==
  
Let </math>y=(x-a)^2+(x-b)^2, a, b<math> constants. For what value of </math>x<math> is </math>y<math> a minimum?
+
Let <math>y=(x-a)^2+(x-b)^2, a, b</math> constants. For what value of <math>x</math> is <math>y</math> a minimum?
  
</math>\textbf{(A)}\ \frac{a+b}{2} \qquad
+
<math>\textbf{(A)}\ \frac{a+b}{2} \qquad
 
\textbf{(B)}\ a+b \qquad
 
\textbf{(B)}\ a+b \qquad
 
\textbf{(C)}\ \sqrt{ab} \qquad
 
\textbf{(C)}\ \sqrt{ab} \qquad
 
\textbf{(D)}\ \sqrt{\frac{a^2+b^2}{2}}\qquad
 
\textbf{(D)}\ \sqrt{\frac{a^2+b^2}{2}}\qquad
\textbf{(E)}\ \frac{a+b}{2ab} <math>  
+
\textbf{(E)}\ \frac{a+b}{2ab} </math>  
  
 
[[1964 AHSME Problems/Problem 24|Solution]]
 
[[1964 AHSME Problems/Problem 24|Solution]]
Line 307: Line 310:
 
== Problem 25==
 
== Problem 25==
  
The set of values of </math>m<math> for which </math>x^2+3xy+x+my-m<math> has two factors, with integer coefficients, which are linear in </math>x<math> and </math>y<math>, is precisely:
+
The set of values of <math>m</math> for which <math>x^2+3xy+x+my-m</math> has two factors, with integer coefficients, which are linear in <math>x</math> and <math>y</math>, is precisely:
  
</math>\textbf{(A)}\ 0, 12, -12\qquad
+
<math>\textbf{(A)}\ 0, 12, -12\qquad
 
\textbf{(B)}\ 0, 12\qquad
 
\textbf{(B)}\ 0, 12\qquad
 
\textbf{(C)}\ 12, -12\qquad
 
\textbf{(C)}\ 12, -12\qquad
 
\textbf{(D)}\ 12\qquad
 
\textbf{(D)}\ 12\qquad
\textbf{(E)}\ 0  <math>   
+
\textbf{(E)}\ 0  </math>   
  
 
[[1964 AHSME Problems/Problem 25|Solution]]
 
[[1964 AHSME Problems/Problem 25|Solution]]
Line 319: Line 322:
 
== Problem 26==
 
== Problem 26==
  
In a ten-mile race </math>First<math> beats </math>Second<math> by </math>2<math> miles and </math>First<math> beats </math>Third<math> by </math>4<math> miles.  
+
In a ten-mile race <math>\textit{First}</math> beats <math>\textit{Second}</math> by <math>2</math> miles and  
If the runners maintain constant speeds throughout the race, by how many miles does </math>Second<math> beat </math>Third<math>?
+
<math>\textit{First}</math> beats <math>\textit{Third}</math> by <math>4</math> miles.  
 +
If the runners maintain constant speeds throughout the race,  
 +
by how many miles does <math>\textit{Second}</math> beat <math>\textit{Third}</math>?
  
</math>\textbf{(A)}\ 2\qquad
+
<math>\textbf{(A)}\ 2\qquad
 
\textbf{(B)}\ 2\frac{1}{4}\qquad
 
\textbf{(B)}\ 2\frac{1}{4}\qquad
 
\textbf{(C)}\ 2\frac{1}{2}\qquad
 
\textbf{(C)}\ 2\frac{1}{2}\qquad
 
\textbf{(D)}\ 2\frac{3}{4}\qquad
 
\textbf{(D)}\ 2\frac{3}{4}\qquad
\textbf{(E)}\ 3 <math>
+
\textbf{(E)}\ 3 </math>
  
 
[[1964 AHSME Problems/Problem 26|Solution]]
 
[[1964 AHSME Problems/Problem 26|Solution]]
Line 332: Line 337:
 
== Problem 27==
 
== Problem 27==
  
If </math>x<math> is a real number and </math>|x-4|+|x-3|<a<math> where </math>a>0<math>, then:
+
If <math>x</math> is a real number and <math>|x-4|+|x-3|<a</math> where <math>a>0</math>, then:
  
</math>\textbf{(A)}\ 0<a<.01\qquad
+
<math>\textbf{(A)}\ 0<a<.01\qquad
 
\textbf{(B)}\ .01<a<1 \qquad
 
\textbf{(B)}\ .01<a<1 \qquad
\textbf{(C)}\ 0<a<1\qquad
+
\textbf{(C)}\ 0<a<1\qquad \\
 
\textbf{(D)}\ 0<a \le 1\qquad
 
\textbf{(D)}\ 0<a \le 1\qquad
\textbf{(E)}\ a>1    <math>  
+
\textbf{(E)}\ a>1    </math>  
  
 
[[1964 AHSME Problems/Problem 27|Solution]]
 
[[1964 AHSME Problems/Problem 27|Solution]]
Line 344: Line 349:
 
== Problem 28==
 
== Problem 28==
  
The sum of </math>n<math> terms of an arithmetic progression is </math>153<math>, and the common difference is </math>2<math>.  
+
The sum of <math>n</math> terms of an arithmetic progression is <math>153</math>, and the common difference is <math>2</math>.  
If the first term is an integer, and </math>n>1<math>, then the number of possible values for </math>n<math> is:
+
If the first term is an integer, and <math>n>1</math>, then the number of possible values for <math>n</math> is:
  
</math>\textbf{(A)}\ 2\qquad
+
<math>\textbf{(A)}\ 2\qquad
 
\textbf{(B)}\ 3\qquad
 
\textbf{(B)}\ 3\qquad
 
\textbf{(C)}\ 4\qquad
 
\textbf{(C)}\ 4\qquad
 
\textbf{(D)}\ 5\qquad
 
\textbf{(D)}\ 5\qquad
\textbf{(E)}\ 6  <math>   
+
\textbf{(E)}\ 6  </math>   
  
 
[[1964 AHSME Problems/Problem 28|Solution]]
 
[[1964 AHSME Problems/Problem 28|Solution]]
Line 357: Line 362:
 
== Problem 29==
 
== Problem 29==
  
In this figure </math>\angle RFS = \angle FDR, FD = 4<math> inches, </math>DR = 6<math> inches, </math>FR = 5<math> inches, </math>FS = 7\dfrac{1}{2}<math> inches.  
+
In this figure <math>\angle RFS = \angle FDR, FD = 4</math> inches, <math>DR = 6</math> inches, <math>FR = 5</math> inches, <math>FS = 7\tfrac{1}{2}</math> inches.  
The length of </math>RS$, in inches, is:
+
The length of <math>RS</math>, in inches, is:
  
 
<asy>
 
<asy>
Line 379: Line 384:
 
<math>\textbf{(A)}\ \text{undetermined} \qquad
 
<math>\textbf{(A)}\ \text{undetermined} \qquad
 
\textbf{(B)}\ 4\qquad
 
\textbf{(B)}\ 4\qquad
\textbf{(C)}\ 5\dfrac{1}{2} \qquad
+
\textbf{(C)}\ 5\tfrac{1}{2} \qquad
 
\textbf{(D)}\ 6\qquad
 
\textbf{(D)}\ 6\qquad
\textbf{(E)}\ 6\frac{1}{4} </math>
+
\textbf{(E)}\ 6\tfrac{1}{4} </math>
  
 
[[1964 AHSME Problems/Problem 29|Solution]]
 
[[1964 AHSME Problems/Problem 29|Solution]]
Line 417: Line 422:
 
\textbf{(B)}\ a+b+c+d\text{ must equal zero }\qquad
 
\textbf{(B)}\ a+b+c+d\text{ must equal zero }\qquad
 
\textbf{(C)}\ \text{either }a=c\text{ or }a+b+c+d=0,\text{ or both} \qquad
 
\textbf{(C)}\ \text{either }a=c\text{ or }a+b+c+d=0,\text{ or both} \qquad
\textbf{(D)}\ a+b+c+d\neq 0\text{ if }a=c \qquad
+
\textbf{(D)}\ a+b+c+d\neq 0\text{ if }a=c \qquad \\
 
\textbf{(E)}\ a(b+c+d)=c(a+b+d)    </math>  
 
\textbf{(E)}\ a(b+c+d)=c(a+b+d)    </math>  
  
Line 456: Line 461:
 
<math>\textbf{(A)}\ 1+i\qquad
 
<math>\textbf{(A)}\ 1+i\qquad
 
\textbf{(B)}\ \frac{1}{2}(n+2) \qquad
 
\textbf{(B)}\ \frac{1}{2}(n+2) \qquad
\textbf{(C)}\ \frac{1}{2}(n+2-ni) \qquad
+
\textbf{(C)}\ \frac{1}{2}(n+2-ni) \qquad \\
 
\textbf{(D)}\ \frac{1}{2}[(n+1)(1-i)+2]\qquad
 
\textbf{(D)}\ \frac{1}{2}[(n+1)(1-i)+2]\qquad
 
\textbf{(E)}\ \frac{1}{8}(n^2+8-4ni) </math>     
 
\textbf{(E)}\ \frac{1}{8}(n^2+8-4ni) </math>     
Line 464: Line 469:
 
== Problem 35==
 
== Problem 35==
  
The sides of a triangle are of lengths <math>13, 14, and 15</math>. The altitudes of the triangle meet at point <math>H</math>.  
+
The sides of a triangle are of lengths <math>13, 14,</math> and <math>15</math>. The altitudes of the triangle meet at point <math>H</math>.  
 
If <math>AD</math> is the altitude to the side length <math>14</math>, what is the ratio <math>HD:HA</math>?     
 
If <math>AD</math> is the altitude to the side length <math>14</math>, what is the ratio <math>HD:HA</math>?     
  
Line 565: Line 570:
 
== Problem 40==
 
== Problem 40==
  
A watch loses <math>2\frac{1}{2}</math> minutes per day. It is set right at <math>1</math> P.M. on March <math>15</math>.  
+
A watch loses <math>2\tfrac{1}{2}</math> minutes per day. It is set right at <math>1</math> P.M. on March <math>15</math>.  
 
Let <math>n</math> be the positive correction, in minutes, to be added to the time shown by the watch at a given time.  
 
Let <math>n</math> be the positive correction, in minutes, to be added to the time shown by the watch at a given time.  
 
When the watch shows <math>9</math> A.M. on March <math>21</math>, <math>n</math> equals:
 
When the watch shows <math>9</math> A.M. on March <math>21</math>, <math>n</math> equals:
Line 576: Line 581:
 
    
 
    
 
[[1964 AHSME Problems/Problem 40|Solution]]
 
[[1964 AHSME Problems/Problem 40|Solution]]
 +
 +
 +
 +
== See also ==
 +
 +
* [[AMC 12 Problems and Solutions]]
 +
* [[Mathematics competition resources]]
 +
 +
{{AHSME 40p box|year=1964|before=[[1963 AHSME|1963 AHSC]]|after=[[1965 AHSME|1965 AHSC]]}} 
 +
 +
{{MAA Notice}}

Latest revision as of 14:16, 20 February 2020

1964 AHSC (Answer Key)
Printable versions: WikiAoPS ResourcesPDF

Instructions

  1. This is a 40-question, multiple choice test. Each question is followed by answers marked A, B, C, D and E. Only one of these is correct.
  2. You will receive ? points for each correct answer, ? points for each problem left unanswered, and ? points for each incorrect answer.
  3. No aids are permitted other than scratch paper, graph paper, ruler, compass, protractor and erasers.
  4. Figures are not necessarily drawn to scale.
  5. You will have ? minutes working time to complete the test.
1 2 3 4 5 6 7 8 9 10 11 12 13 14 15 16 17 18 19 20 21 22 23 24 25 26 27 28 29 30 31 32 33 34 35 36 37 38 39 40

Problem 1

What is the value of $[\log_{10}(5\log_{10}100)]^2$?

$\textbf{(A)}\ \log_{10}50 \qquad \textbf{(B)}\ 25\qquad \textbf{(C)}\ 10 \qquad \textbf{(D)}\ 2\qquad \textbf{(E)}\ 1$

Solution

Problem 2

The graph of $x^2-4y^2=0$ is:

$\textbf{(A)}\ \text{a parabola} \qquad \textbf{(B)}\ \text{an ellipse} \qquad \textbf{(C)}\ \text{a pair of straight lines}\qquad \\ \textbf{(D)}\ \text{a point}\qquad \textbf{(E)}\ \text{None of these}$

Solution

Problem 3

When a positive integer $x$ is divided by a positive integer $y$, the quotient is $u$ and the remainder is $v$, where $u$ and $v$ are integers. What is the remainder when $x+2uy$ is divided by $y$?

$\textbf{(A)}\ 0 \qquad \textbf{(B)}\ 2u \qquad \textbf{(C)}\ 3u \qquad \textbf{(D)}\ v \qquad \textbf{(E)}\ 2v$

Solution

Problem 4

The expression

\[\frac{P+Q}{P-Q}-\frac{P-Q}{P+Q}\]

where $P=x+y$ and $Q=x-y$, is equivalent to:

$\textbf{(A)}\ \frac{x^2-y^2}{xy}\qquad \textbf{(B)}\ \frac{x^2-y^2}{2xy}\qquad \textbf{(C)}\ 1 \qquad \textbf{(D)}\ \frac{x^2+y^2}{xy}\qquad \textbf{(E)}\ \frac{x^2+y^2}{2xy}$

Solution

Problem 5

If $y$ varies directly as $x$, and if $y=8$ when $x=4$, the value of $y$ when $x=-8$ is:

$\textbf{(A)}\ -16 \qquad \textbf{(B)}\ -4 \qquad \textbf{(C)}\ -2 \qquad \textbf{(D)}\ 4k, k= \pm1, \pm2, \dots \qquad \\ \textbf{(E)}\ 16k, k=\pm1,\pm2,\dots$

Solution

Problem 6

If $x, 2x+2, 3x+3, \dots$ are in geometric progression, the fourth term is:

$\textbf{(A)}\ -27 \qquad \textbf{(B)}\ -13\frac{1}{2} \qquad \textbf{(C)}\ 12\qquad \textbf{(D)}\ 13\frac{1}{2}\qquad \textbf{(E)}\ 27$

Solution

Problem 7

Let n be the number of real values of $p$ for which the roots of $x^2-px+p=0$ are equal. Then n equals:

$\textbf{(A)}\ 0 \qquad \textbf{(B)}\ 1 \qquad \textbf{(C)}\ 2 \qquad \textbf{(D)}\ \text{a finite number greater than 2}\qquad \textbf{(E)}\ \infty$

Solution

Problem 8

The smaller root of the equation $\left(x-\frac{3}{4}\right)\left(x-\frac{3}{4}\right)+\left(x-\frac{3}{4}\right)\left(x-\frac{1}{2}\right) =0$ is:

$\textbf{(A)}\ -\frac{3}{4}\qquad \textbf{(B)}\ \frac{1}{2}\qquad \textbf{(C)}\ \frac{5}{8}\qquad \textbf{(D)}\ \frac{3}{4}\qquad \textbf{(E)}\ 1$

Solution

Problem 9

A jobber buys an article at $$24$ less $12\frac{1}{2}\%$. He then wishes to sell the article at a gain of $33\frac{1}{3}\%$ of his cost after allowing a $20\%$ discount on his marked price. At what price, in dollars, should the article be marked?

$\textbf{(A)}\ 25.20 \qquad \textbf{(B)}\ 30.00 \qquad \textbf{(C)}\ 33.60 \qquad \textbf{(D)}\ 40.00 \qquad \textbf{(E)}\ \text{none of these}$

Solution

Problem 10

Given a square side of length $s$. On a diagonal as base a triangle with three unequal sides is constructed so that its area equals that of the square. The length of the altitude drawn to the base is:

$\textbf{(A)}\ s\sqrt{2} \qquad \textbf{(B)}\ s/\sqrt{2} \qquad \textbf{(C)}\ 2s \qquad \textbf{(D)}\ 2\sqrt{s} \qquad \textbf{(E)}\ 2/\sqrt{s}$

Solution

Problem 11

Given $2^x=8^{y+1}$ and $9^y=3^{x-9}$, find the value of $x+y$

$\textbf{(A)}\ 18 \qquad \textbf{(B)}\ 21 \qquad \textbf{(C)}\ 24 \qquad \textbf{(D)}\ 27 \qquad \textbf{(E)}\ 30$

Solution

Problem 12

Which of the following is the negation of the statement: For all $x$ of a certain set, $x^2>0$?

$\textbf{(A)}\ \text{For all x}, x^2 < 0\qquad \textbf{(B)}\ \text{For all x}, x^2 \le 0\qquad \textbf{(C)}\ \text{For no x}, x^2>0\qquad \\ \textbf{(D)}\ \text{For some x}, x^2>0\qquad \textbf{(E)}\ \text{For some x}, x^2 \le 0$

Solution

Problem 13

A circle is inscribed in a triangle with side lengths $8, 13$, and $17$. Let the segments of the side of length $8$, made by a point of tangency, be $r$ and $s$, with $r<s$. What is the ratio $r:s$?

$\textbf{(A)}\ 1:3 \qquad \textbf{(B)}\ 2:5 \qquad \textbf{(C)}\ 1:2 \qquad \textbf{(D)}\ 2:3 \qquad \textbf{(E)}\ 3:4$

Solution

Problem 14

A farmer bought $749$ sheep. He sold $700$ of them for the price paid for the $749$ sheep. The remaining $49$ sheep were sold at the same price per head as the other $700$. Based on the cost, the percent gain on the entire transaction is:

$\textbf{(A)}\ 6.5 \qquad \textbf{(B)}\ 6.75 \qquad \textbf{(C)}\ 7 \qquad \textbf{(D)}\ 7.5 \qquad \textbf{(E)}\ 8$

Solution

Problem 15

A line through the point $(-a,0)$ cuts from the second quadrant a triangular region with area $T$. The equation of the line is:

$\textbf{(A)}\ 2Tx+a^2y+2aT=0 \qquad \textbf{(B)}\ 2Tx-a^2y+2aT=0 \qquad \textbf{(C)}\ 2Tx+a^2y-2aT=0 \qquad \\ \textbf{(D)}\ 2Tx-a^2y-2aT=0 \qquad \textbf{(E)}\ \text{none of these}$

Solution

Problem 16

Let $f(x)=x^2+3x+2$ and let $S$ be the set of integers $\{0, 1, 2, \dots , 25 \}$. The number of members $s$ of $S$ such that $f(s)$ has remainder zero when divided by $6$ is:

$\textbf{(A)}\ 25\qquad \textbf{(B)}\ 22\qquad \textbf{(C)}\ 21\qquad \textbf{(D)}\ 18 \qquad \textbf{(E)}\ 17$

Solution

Problem 17

Given the distinct points $P(x_1, y_1), Q(x_2, y_2)$ and $R(x_1+x_2, y_1+y_2)$. Line segments are drawn connecting these points to each other and to the origin $O$. Of the three possibilities: (1) parallelogram (2) straight line (3) trapezoid, figure $OPRQ$, depending upon the location of the points $P, Q$, and $R$, can be:

$\textbf{(A)}\ \text{(1) only}\qquad \textbf{(B)}\ \text{(2) only}\qquad \textbf{(C)}\ \text{(3) only}\qquad \textbf{(D)}\ \text{(1) or (2) only}\qquad \textbf{(E)}\ \text{all three}$

Solution

Problem 18

Let $n$ be the number of pairs of values of $b$ and $c$ such that $3x+by+c=0$ and $cx-2y+12=0$ have the same graph. Then $n$ is:

$\textbf{(A)}\ 0\qquad \textbf{(B)}\ 1\qquad \textbf{(C)}\ 2\qquad \textbf{(D)}\ \text{finite but more than 2}\qquad \textbf{(E)}\ \infty$

Solution

Problem 19

If $2x-3y-z=0$ and $x+3y-14z=0, z \neq 0$, the numerical value of $\frac{x^2+3xy}{y^2+z^2}$ is:

$\textbf{(A)}\ 7\qquad \textbf{(B)}\ 2\qquad \textbf{(C)}\ 0\qquad \textbf{(D)}\ -20/17\qquad \textbf{(E)}\ -2$

Solution

Problem 20

The sum of the numerical coefficients of all the terms in the expansion of $(x-2y)^{18}$ is:

$\textbf{(A)}\ 0\qquad \textbf{(B)}\ 1\qquad \textbf{(C)}\ 19\qquad \textbf{(D)}\ -1\qquad \textbf{(E)}\ -19$

Solution

Problem 21

If $\log_{b^2}x+\log_{x^2}b=1, b>0, b \neq 1, x \neq 1$, then $x$ equals:

$\textbf{(A)}\ 1/b^2 \qquad \textbf{(B)}\ 1/b \qquad \textbf{(C)}\ b^2 \qquad \textbf{(D)}\ b \qquad \textbf{(E)}\ \sqrt{b}$

Solution

Problem 22

Given parallelogram $ABCD$ with $E$ the midpoint of diagonal $BD$. Point $E$ is connected to a point $F$ in $DA$ so that $DF=\frac{1}{3}DA$. What is the ratio of the area of $\triangle DFE$ to the area of quadrilateral $ABEF$?

$\textbf{(A)}\ 1:2 \qquad \textbf{(B)}\ 1:3 \qquad \textbf{(C)}\ 1:5 \qquad \textbf{(D)}\ 1:6 \qquad \textbf{(E)}\ 1:7$

Solution

Problem 23

Two numbers are such that their difference, their sum, and their product are to one another as $1:7:24$. The product of the two numbers is:

$\textbf{(A)}\ 6\qquad \textbf{(B)}\ 12\qquad \textbf{(C)}\ 24\qquad \textbf{(D)}\ 48\qquad \textbf{(E)}\ 96$

Solution

Problem 24

Let $y=(x-a)^2+(x-b)^2, a, b$ constants. For what value of $x$ is $y$ a minimum?

$\textbf{(A)}\ \frac{a+b}{2} \qquad \textbf{(B)}\ a+b \qquad \textbf{(C)}\ \sqrt{ab} \qquad \textbf{(D)}\ \sqrt{\frac{a^2+b^2}{2}}\qquad \textbf{(E)}\ \frac{a+b}{2ab}$

Solution

Problem 25

The set of values of $m$ for which $x^2+3xy+x+my-m$ has two factors, with integer coefficients, which are linear in $x$ and $y$, is precisely:

$\textbf{(A)}\ 0, 12, -12\qquad \textbf{(B)}\ 0, 12\qquad \textbf{(C)}\ 12, -12\qquad \textbf{(D)}\ 12\qquad \textbf{(E)}\ 0$

Solution

Problem 26

In a ten-mile race $\textit{First}$ beats $\textit{Second}$ by $2$ miles and $\textit{First}$ beats $\textit{Third}$ by $4$ miles. If the runners maintain constant speeds throughout the race, by how many miles does $\textit{Second}$ beat $\textit{Third}$?

$\textbf{(A)}\ 2\qquad \textbf{(B)}\ 2\frac{1}{4}\qquad \textbf{(C)}\ 2\frac{1}{2}\qquad \textbf{(D)}\ 2\frac{3}{4}\qquad \textbf{(E)}\ 3$

Solution

Problem 27

If $x$ is a real number and $|x-4|+|x-3|<a$ where $a>0$, then:

$\textbf{(A)}\ 0<a<.01\qquad \textbf{(B)}\ .01<a<1 \qquad \textbf{(C)}\ 0<a<1\qquad \\ \textbf{(D)}\ 0<a \le 1\qquad \textbf{(E)}\ a>1$

Solution

Problem 28

The sum of $n$ terms of an arithmetic progression is $153$, and the common difference is $2$. If the first term is an integer, and $n>1$, then the number of possible values for $n$ is:

$\textbf{(A)}\ 2\qquad \textbf{(B)}\ 3\qquad \textbf{(C)}\ 4\qquad \textbf{(D)}\ 5\qquad \textbf{(E)}\ 6$

Solution

Problem 29

In this figure $\angle RFS = \angle FDR, FD = 4$ inches, $DR = 6$ inches, $FR = 5$ inches, $FS = 7\tfrac{1}{2}$ inches. The length of $RS$, in inches, is:

[asy] import olympiad; pair F,R,S,D; F=origin;  R=5*dir(aCos(9/16)); S=(7.5,0); D=4*dir(aCos(9/16)+aCos(1/8)); label("$F$",F,SW);label("$R$",R,N); label("$S$",S,SE); label("$D$",D,W); label("$7\frac{1}{2}$",(F+S)/2.5,SE); label("$4$",midpoint(F--D),SW); label("$5$",midpoint(F--R),W); label("$6$",midpoint(D--R),N); draw(F--D--R--F--S--R);  markscalefactor=0.1; draw(anglemark(S,F,R)); draw(anglemark(F,D,R)); //Credit to throwaway1489 for the diagram[/asy]

$\textbf{(A)}\ \text{undetermined} \qquad \textbf{(B)}\ 4\qquad \textbf{(C)}\ 5\tfrac{1}{2} \qquad \textbf{(D)}\ 6\qquad \textbf{(E)}\ 6\tfrac{1}{4}$

Solution

Problem 30

If $(7+4\sqrt{3})x^2+(2+\sqrt{3})x-2=0$, the larger root minus the smaller root is:

$\textbf{(A)}\ -2+3\sqrt{3}\qquad \textbf{(B)}\ 2-\sqrt{3}\qquad \textbf{(C)}\ 6+3\sqrt{3}\qquad \textbf{(D)}\ 6-3\sqrt{3}\qquad \textbf{(E)}\ 3\sqrt{3}+2$

Solution

Problem 31

Let $f(n)=\dfrac{5+3\sqrt{5}}{10}\left(\dfrac{1+\sqrt{5}}{2}\right)^n+\dfrac{5-3\sqrt{5}}{10}\left(\dfrac{1-\sqrt{5}}{2}\right)^n$. Then $f(n+1)-f(n-1)$, expressed in terms of $f(n)$, equals:

$\textbf{(A)}\ \dfrac{1}{2}f(n) \qquad \textbf{(B)}\ f(n)\qquad \textbf{(C)}\ 2f(n)+1 \qquad \textbf{(D)}\ f^2(n)\qquad \textbf{(E)}\ \frac{1}{2}(f^2(n)-1)$

Solution

Problem 32

If $\dfrac{a+b}{b+c}=\dfrac{c+d}{d+a}$, then:

$\textbf{(A)}\ a\text{ must equal }c \qquad \textbf{(B)}\ a+b+c+d\text{ must equal zero }\qquad \textbf{(C)}\ \text{either }a=c\text{ or }a+b+c+d=0,\text{ or both} \qquad \textbf{(D)}\ a+b+c+d\neq 0\text{ if }a=c \qquad \\ \textbf{(E)}\ a(b+c+d)=c(a+b+d)$

Solution

Problem 33

$P$ is a point interior to rectangle $ABCD$ and such that $PA=3$ inches, $PD=4$ inches, and $PC=5$ inches. Then $PB$, in inches, equals:

$\textbf{(A) }2\sqrt{3}\qquad \textbf{(B) }3\sqrt{2}\qquad \textbf{(C) }3\sqrt{3}\qquad \textbf{(D) }4\sqrt{2}\qquad \textbf{(E) }2$

[asy] draw((0,0)--(6.5,0)--(6.5,4.5)--(0,4.5)--cycle); draw((2.5,1.5)--(0,0)); draw((2.5,1.5)--(0,4.5)); draw((2.5,1.5)--(6.5,4.5)); draw((2.5,1.5)--(6.5,0),linetype("8 8")); label("$A$",(0,0),dir(-135)); label("$B$",(6.5,0),dir(-45)); label("$C$",(6.5,4.5),dir(45)); label("$D$",(0,4.5),dir(135)); label("$P$",(2.5,1.5),dir(-90)); label("$3$",(1.25,0.75),dir(120)); label("$4$",(1.25,3),dir(35)); label("$5$",(4.5,3),dir(120)); //Credit to bobthesmartypants for the diagram[/asy]

Solution

Problem 34

If $n$ is a multiple of $4$, the sum $s=1+2i+3i^2+ \ldots +(n+1)i^{n}$, where $i=\sqrt{-1}$, equals:

$\textbf{(A)}\ 1+i\qquad \textbf{(B)}\ \frac{1}{2}(n+2) \qquad \textbf{(C)}\ \frac{1}{2}(n+2-ni) \qquad \\ \textbf{(D)}\ \frac{1}{2}[(n+1)(1-i)+2]\qquad \textbf{(E)}\ \frac{1}{8}(n^2+8-4ni)$

Solution

Problem 35

The sides of a triangle are of lengths $13, 14,$ and $15$. The altitudes of the triangle meet at point $H$. If $AD$ is the altitude to the side length $14$, what is the ratio $HD:HA$?

Solution

Problem 36

In this figure the radius of the circle is equal to the altitude of the equilateral triangle $ABC$. The circle is made to roll along the side $AB$, remaining tangent to it at a variable point $T$ and intersecting lines $AC$ and $BC$ in variable points $M$ and $N$, respectively. Let $n$ be the number of degrees in arc $MTN$. Then $n$, for all permissible positions of the circle:

$\textbf{(A) }\text{varies from }30^{\circ}\text{ to }90^{\circ}\quad \textbf{(B) }\text{varies from }30^{\circ}\text{ to }60^{\circ} \\ \textbf{(C) }\text{varies from }60^{\circ}\text{ to }90^{\circ} \quad \textbf{(D) }\text{remains constant at }30^{\circ}\quad \textbf{(E) }\text{remains constant at }60^{\circ}$

[asy] pair A = (0,0), B = (1,0), C = dir(60), T = (2/3,0); pair M = intersectionpoint(A--C,Circle((2/3,sqrt(3)/2),sqrt(3)/2)), N = intersectionpoint(B--C,Circle((2/3,sqrt(3)/2),sqrt(3)/2)); draw((0,0)--(1,0)--dir(60)--cycle); draw(Circle((2/3,sqrt(3)/2),sqrt(3)/2)); label("$A$",A,dir(210)); label("$B$",B,dir(-30)); label("$C$",C,dir(90)); label("$M$",M,dir(190)); label("$N$",N,dir(75)); label("$T$",T,dir(-90)); //Credit to bobthesmartypants for the diagram[/asy]

Solution

Problem 37

Given two positive number $a, b$ such that $a<b$ , let $A.M.$ be their arithmetic mean and let $G.M.$ be their positive geometric mean. Then $A.M.$ minus $G.M.$ is always less than:

$\textbf{(A) }\dfrac{(b+a)^2}{ab}\qquad \textbf{(B) }\dfrac{(b+a)^2}{8b}\qquad \textbf{(C) }\dfrac{(b-a)^2}{ab} \textbf{(D) }\dfrac{(b-a)^2}{8a}\qquad  \textbf{(E) }\dfrac{(b-a)^2}{8b}$

Solution

Problem 38

The sides $PQ$ and $PR$ of $\triangle PQR$ are respectively of lengths $4$ inches, and $7$ inches. The median $PM$ is $3\frac{1}{2}$ inches. Then $QR$, in inches, is:

$\textbf{(A) }6\qquad \textbf{(B) }7\qquad \textbf{(C) }8\qquad \textbf{(D) }9\qquad  \textbf{(E) }10$

Solution

Problem 39

The magnitudes of the sides of $\triangle ABC$ are $a, b$, and $c$, as shown, with $c\le b\le a$. Through interior point $P$ and the vertices $A, B, C$, lines are drawn meeting the opposite sides in $A', B', C'$, respectively. Let $s=AA'+BB'+CC'$. Then, for all positions of point $P$, $s$ is less than:

$\textbf{(A) }2a+b\qquad \textbf{(B) }2a+c\qquad \textbf{(C) }2b+c\qquad \textbf{(D) }a+2b\qquad  \textbf{(E) } a+b+c$


[asy] import math; defaultpen(fontsize(11pt)); pair A = (0,0), B = (1,3), C = (5,0), P = (1.5,1); pair X = extension(B,C,A,P), Y = extension(A,C,B,P), Z = extension(A,B,C,P); draw(A--B--C--cycle); draw(A--X); draw(B--Y); draw(C--Z); dot(P); dot(A); dot(B); dot(C); label("$A$",A,dir(210)); label("$B$",B,dir(90)); label("$C$",C,dir(-30)); label("$A'$",X,dir(-100)); label("$B'$",Y,dir(65)); label("$C'$",Z,dir(20)); label("$P$",P,dir(70)); label("$a$",X,dir(80)); label("$b$",Y,dir(-90)); label("$c$",Z,dir(110)); //Credit to bobthesmartypants for the diagram[/asy]

Solution

Problem 40

A watch loses $2\tfrac{1}{2}$ minutes per day. It is set right at $1$ P.M. on March $15$. Let $n$ be the positive correction, in minutes, to be added to the time shown by the watch at a given time. When the watch shows $9$ A.M. on March $21$, $n$ equals:

$\textbf{(A) }14\frac{14}{23}\qquad \textbf{(B) }14\frac{1}{14}\qquad \textbf{(C) }13\frac{101}{115}\qquad \textbf{(D) }13\frac{83}{115}\qquad \textbf{(E) }13\frac{13}{23}$

Solution


See also

1964 AHSC (ProblemsAnswer KeyResources)
Preceded by
1963 AHSC
Followed by
1965 AHSC
1 2 3 4 5 6 7 8 9 10 11 12 13 14 15 16 17 18 19 20 21 22 23 24 25 26 27 28 29 30 31 32 33 34 35 36 37 38 39 40
All AHSME Problems and Solutions


The problems on this page are copyrighted by the Mathematical Association of America's American Mathematics Competitions. AMC logo.png